What is wrong with this false proof of $pi=0$?












4












$begingroup$


Consider the integral
$$I=int_{-1}^{1}frac{1}{x^2+1}mathrm{d}x$$
Now, from the standard integral results we know,
$$intfrac{1}{x^2+1}mathrm{d}x=arctan(x)+c$$
So,
$$int_{-1}^{1}frac{1}{x^2+1}mathrm{d}x=arctan(1)-arctan(-1)=frac{pi}{4}-(-frac{pi}{4})=frac{pi}{2}$$
Now, if you are bored and just doing random things you might evaluate this integral in a roundabout way by the substitution $u=dfrac{1}{x}$.



This gives the bound for the integrals as $u=dfrac{1}{-1}=-1$ to $u=dfrac{1}{1}=1$ and the differential becomes $mathrm{d}{x}=dfrac{-1}{u^2}mathrm{d}{u}$. So, the integral becomes
$$int_{-1}^{1}frac{1}{frac{1}{u^2}+1}frac{-1}{u^2}mathrm{d}u = -int_{-1}^{1}frac{1}{u^2+1}mathrm{d}u=-I$$.



Now, $$I=-I implies 2I=0implies pi=0$$.



So, there is your false proof.



I think that the problem in the proof comes when we do the substitution $x=dfrac{1}{u}$. But, I dunno what it is. Does it have to do something with the continuity of the substitution, or is it something else.










share|cite|improve this question











$endgroup$








  • 1




    $begingroup$
    Think about the bounds. What happens when $xto0$?
    $endgroup$
    – Peter Foreman
    yesterday










  • $begingroup$
    @PeterForeman So, if the substituted variable becomes discontinuous at some point where the original variable was continuous so we cant proceed with the integration with substitution. Am I correct to infer that?
    $endgroup$
    – user350331
    yesterday






  • 1




    $begingroup$
    You can change limits, if variable is undefined somewhere. 1/x is discontinuous (to say more precisely) in range [-1,1]. For this you have to seperate into two integrals $int_{-1}^{0^-} + int_{0^+}^1$
    $endgroup$
    – Tojrah
    yesterday










  • $begingroup$
    BTW, with TeX and MathJax, you want the punctuation inside the double dollar sign, since that ends the line. Otherwise, you end up with the punctuation all by itself on the next line. (With a single dollar sign, you want the punctuation outside, since the punctuation isn't part of the math.)
    $endgroup$
    – Teepeemm
    yesterday
















4












$begingroup$


Consider the integral
$$I=int_{-1}^{1}frac{1}{x^2+1}mathrm{d}x$$
Now, from the standard integral results we know,
$$intfrac{1}{x^2+1}mathrm{d}x=arctan(x)+c$$
So,
$$int_{-1}^{1}frac{1}{x^2+1}mathrm{d}x=arctan(1)-arctan(-1)=frac{pi}{4}-(-frac{pi}{4})=frac{pi}{2}$$
Now, if you are bored and just doing random things you might evaluate this integral in a roundabout way by the substitution $u=dfrac{1}{x}$.



This gives the bound for the integrals as $u=dfrac{1}{-1}=-1$ to $u=dfrac{1}{1}=1$ and the differential becomes $mathrm{d}{x}=dfrac{-1}{u^2}mathrm{d}{u}$. So, the integral becomes
$$int_{-1}^{1}frac{1}{frac{1}{u^2}+1}frac{-1}{u^2}mathrm{d}u = -int_{-1}^{1}frac{1}{u^2+1}mathrm{d}u=-I$$.



Now, $$I=-I implies 2I=0implies pi=0$$.



So, there is your false proof.



I think that the problem in the proof comes when we do the substitution $x=dfrac{1}{u}$. But, I dunno what it is. Does it have to do something with the continuity of the substitution, or is it something else.










share|cite|improve this question











$endgroup$








  • 1




    $begingroup$
    Think about the bounds. What happens when $xto0$?
    $endgroup$
    – Peter Foreman
    yesterday










  • $begingroup$
    @PeterForeman So, if the substituted variable becomes discontinuous at some point where the original variable was continuous so we cant proceed with the integration with substitution. Am I correct to infer that?
    $endgroup$
    – user350331
    yesterday






  • 1




    $begingroup$
    You can change limits, if variable is undefined somewhere. 1/x is discontinuous (to say more precisely) in range [-1,1]. For this you have to seperate into two integrals $int_{-1}^{0^-} + int_{0^+}^1$
    $endgroup$
    – Tojrah
    yesterday










  • $begingroup$
    BTW, with TeX and MathJax, you want the punctuation inside the double dollar sign, since that ends the line. Otherwise, you end up with the punctuation all by itself on the next line. (With a single dollar sign, you want the punctuation outside, since the punctuation isn't part of the math.)
    $endgroup$
    – Teepeemm
    yesterday














4












4








4





$begingroup$


Consider the integral
$$I=int_{-1}^{1}frac{1}{x^2+1}mathrm{d}x$$
Now, from the standard integral results we know,
$$intfrac{1}{x^2+1}mathrm{d}x=arctan(x)+c$$
So,
$$int_{-1}^{1}frac{1}{x^2+1}mathrm{d}x=arctan(1)-arctan(-1)=frac{pi}{4}-(-frac{pi}{4})=frac{pi}{2}$$
Now, if you are bored and just doing random things you might evaluate this integral in a roundabout way by the substitution $u=dfrac{1}{x}$.



This gives the bound for the integrals as $u=dfrac{1}{-1}=-1$ to $u=dfrac{1}{1}=1$ and the differential becomes $mathrm{d}{x}=dfrac{-1}{u^2}mathrm{d}{u}$. So, the integral becomes
$$int_{-1}^{1}frac{1}{frac{1}{u^2}+1}frac{-1}{u^2}mathrm{d}u = -int_{-1}^{1}frac{1}{u^2+1}mathrm{d}u=-I$$.



Now, $$I=-I implies 2I=0implies pi=0$$.



So, there is your false proof.



I think that the problem in the proof comes when we do the substitution $x=dfrac{1}{u}$. But, I dunno what it is. Does it have to do something with the continuity of the substitution, or is it something else.










share|cite|improve this question











$endgroup$




Consider the integral
$$I=int_{-1}^{1}frac{1}{x^2+1}mathrm{d}x$$
Now, from the standard integral results we know,
$$intfrac{1}{x^2+1}mathrm{d}x=arctan(x)+c$$
So,
$$int_{-1}^{1}frac{1}{x^2+1}mathrm{d}x=arctan(1)-arctan(-1)=frac{pi}{4}-(-frac{pi}{4})=frac{pi}{2}$$
Now, if you are bored and just doing random things you might evaluate this integral in a roundabout way by the substitution $u=dfrac{1}{x}$.



This gives the bound for the integrals as $u=dfrac{1}{-1}=-1$ to $u=dfrac{1}{1}=1$ and the differential becomes $mathrm{d}{x}=dfrac{-1}{u^2}mathrm{d}{u}$. So, the integral becomes
$$int_{-1}^{1}frac{1}{frac{1}{u^2}+1}frac{-1}{u^2}mathrm{d}u = -int_{-1}^{1}frac{1}{u^2+1}mathrm{d}u=-I$$.



Now, $$I=-I implies 2I=0implies pi=0$$.



So, there is your false proof.



I think that the problem in the proof comes when we do the substitution $x=dfrac{1}{u}$. But, I dunno what it is. Does it have to do something with the continuity of the substitution, or is it something else.







integration fake-proofs






share|cite|improve this question















share|cite|improve this question













share|cite|improve this question




share|cite|improve this question








edited yesterday









YuiTo Cheng

2,43841037




2,43841037










asked yesterday









user350331user350331

1,32321232




1,32321232








  • 1




    $begingroup$
    Think about the bounds. What happens when $xto0$?
    $endgroup$
    – Peter Foreman
    yesterday










  • $begingroup$
    @PeterForeman So, if the substituted variable becomes discontinuous at some point where the original variable was continuous so we cant proceed with the integration with substitution. Am I correct to infer that?
    $endgroup$
    – user350331
    yesterday






  • 1




    $begingroup$
    You can change limits, if variable is undefined somewhere. 1/x is discontinuous (to say more precisely) in range [-1,1]. For this you have to seperate into two integrals $int_{-1}^{0^-} + int_{0^+}^1$
    $endgroup$
    – Tojrah
    yesterday










  • $begingroup$
    BTW, with TeX and MathJax, you want the punctuation inside the double dollar sign, since that ends the line. Otherwise, you end up with the punctuation all by itself on the next line. (With a single dollar sign, you want the punctuation outside, since the punctuation isn't part of the math.)
    $endgroup$
    – Teepeemm
    yesterday














  • 1




    $begingroup$
    Think about the bounds. What happens when $xto0$?
    $endgroup$
    – Peter Foreman
    yesterday










  • $begingroup$
    @PeterForeman So, if the substituted variable becomes discontinuous at some point where the original variable was continuous so we cant proceed with the integration with substitution. Am I correct to infer that?
    $endgroup$
    – user350331
    yesterday






  • 1




    $begingroup$
    You can change limits, if variable is undefined somewhere. 1/x is discontinuous (to say more precisely) in range [-1,1]. For this you have to seperate into two integrals $int_{-1}^{0^-} + int_{0^+}^1$
    $endgroup$
    – Tojrah
    yesterday










  • $begingroup$
    BTW, with TeX and MathJax, you want the punctuation inside the double dollar sign, since that ends the line. Otherwise, you end up with the punctuation all by itself on the next line. (With a single dollar sign, you want the punctuation outside, since the punctuation isn't part of the math.)
    $endgroup$
    – Teepeemm
    yesterday








1




1




$begingroup$
Think about the bounds. What happens when $xto0$?
$endgroup$
– Peter Foreman
yesterday




$begingroup$
Think about the bounds. What happens when $xto0$?
$endgroup$
– Peter Foreman
yesterday












$begingroup$
@PeterForeman So, if the substituted variable becomes discontinuous at some point where the original variable was continuous so we cant proceed with the integration with substitution. Am I correct to infer that?
$endgroup$
– user350331
yesterday




$begingroup$
@PeterForeman So, if the substituted variable becomes discontinuous at some point where the original variable was continuous so we cant proceed with the integration with substitution. Am I correct to infer that?
$endgroup$
– user350331
yesterday




1




1




$begingroup$
You can change limits, if variable is undefined somewhere. 1/x is discontinuous (to say more precisely) in range [-1,1]. For this you have to seperate into two integrals $int_{-1}^{0^-} + int_{0^+}^1$
$endgroup$
– Tojrah
yesterday




$begingroup$
You can change limits, if variable is undefined somewhere. 1/x is discontinuous (to say more precisely) in range [-1,1]. For this you have to seperate into two integrals $int_{-1}^{0^-} + int_{0^+}^1$
$endgroup$
– Tojrah
yesterday












$begingroup$
BTW, with TeX and MathJax, you want the punctuation inside the double dollar sign, since that ends the line. Otherwise, you end up with the punctuation all by itself on the next line. (With a single dollar sign, you want the punctuation outside, since the punctuation isn't part of the math.)
$endgroup$
– Teepeemm
yesterday




$begingroup$
BTW, with TeX and MathJax, you want the punctuation inside the double dollar sign, since that ends the line. Otherwise, you end up with the punctuation all by itself on the next line. (With a single dollar sign, you want the punctuation outside, since the punctuation isn't part of the math.)
$endgroup$
– Teepeemm
yesterday










2 Answers
2






active

oldest

votes


















4












$begingroup$

What you should actually get is that
$$I=int_0^1 frac1{x^2+1}dx+int_{-1}^0 frac1{x^2+1}dx$$
Then the substitution will give
$$I=-int_infty^1 frac1{u^2+1}du-int_{-1}^{-infty} frac1{u^2+1}du$$






share|cite|improve this answer









$endgroup$





















    4












    $begingroup$

    The flaw is in changing the integration interval.



    $$-1le xle1iff frac1xle-1lor frac1xge 1$$






    share|cite|improve this answer









    $endgroup$














      Your Answer








      StackExchange.ready(function() {
      var channelOptions = {
      tags: "".split(" "),
      id: "69"
      };
      initTagRenderer("".split(" "), "".split(" "), channelOptions);

      StackExchange.using("externalEditor", function() {
      // Have to fire editor after snippets, if snippets enabled
      if (StackExchange.settings.snippets.snippetsEnabled) {
      StackExchange.using("snippets", function() {
      createEditor();
      });
      }
      else {
      createEditor();
      }
      });

      function createEditor() {
      StackExchange.prepareEditor({
      heartbeatType: 'answer',
      autoActivateHeartbeat: false,
      convertImagesToLinks: true,
      noModals: true,
      showLowRepImageUploadWarning: true,
      reputationToPostImages: 10,
      bindNavPrevention: true,
      postfix: "",
      imageUploader: {
      brandingHtml: "Powered by u003ca class="icon-imgur-white" href="https://imgur.com/"u003eu003c/au003e",
      contentPolicyHtml: "User contributions licensed under u003ca href="https://creativecommons.org/licenses/by-sa/3.0/"u003ecc by-sa 3.0 with attribution requiredu003c/au003e u003ca href="https://stackoverflow.com/legal/content-policy"u003e(content policy)u003c/au003e",
      allowUrls: true
      },
      noCode: true, onDemand: true,
      discardSelector: ".discard-answer"
      ,immediatelyShowMarkdownHelp:true
      });


      }
      });














      draft saved

      draft discarded


















      StackExchange.ready(
      function () {
      StackExchange.openid.initPostLogin('.new-post-login', 'https%3a%2f%2fmath.stackexchange.com%2fquestions%2f3188675%2fwhat-is-wrong-with-this-false-proof-of-pi-0%23new-answer', 'question_page');
      }
      );

      Post as a guest















      Required, but never shown

























      2 Answers
      2






      active

      oldest

      votes








      2 Answers
      2






      active

      oldest

      votes









      active

      oldest

      votes






      active

      oldest

      votes









      4












      $begingroup$

      What you should actually get is that
      $$I=int_0^1 frac1{x^2+1}dx+int_{-1}^0 frac1{x^2+1}dx$$
      Then the substitution will give
      $$I=-int_infty^1 frac1{u^2+1}du-int_{-1}^{-infty} frac1{u^2+1}du$$






      share|cite|improve this answer









      $endgroup$


















        4












        $begingroup$

        What you should actually get is that
        $$I=int_0^1 frac1{x^2+1}dx+int_{-1}^0 frac1{x^2+1}dx$$
        Then the substitution will give
        $$I=-int_infty^1 frac1{u^2+1}du-int_{-1}^{-infty} frac1{u^2+1}du$$






        share|cite|improve this answer









        $endgroup$
















          4












          4








          4





          $begingroup$

          What you should actually get is that
          $$I=int_0^1 frac1{x^2+1}dx+int_{-1}^0 frac1{x^2+1}dx$$
          Then the substitution will give
          $$I=-int_infty^1 frac1{u^2+1}du-int_{-1}^{-infty} frac1{u^2+1}du$$






          share|cite|improve this answer









          $endgroup$



          What you should actually get is that
          $$I=int_0^1 frac1{x^2+1}dx+int_{-1}^0 frac1{x^2+1}dx$$
          Then the substitution will give
          $$I=-int_infty^1 frac1{u^2+1}du-int_{-1}^{-infty} frac1{u^2+1}du$$







          share|cite|improve this answer












          share|cite|improve this answer



          share|cite|improve this answer










          answered yesterday









          Peter ForemanPeter Foreman

          7,8731320




          7,8731320























              4












              $begingroup$

              The flaw is in changing the integration interval.



              $$-1le xle1iff frac1xle-1lor frac1xge 1$$






              share|cite|improve this answer









              $endgroup$


















                4












                $begingroup$

                The flaw is in changing the integration interval.



                $$-1le xle1iff frac1xle-1lor frac1xge 1$$






                share|cite|improve this answer









                $endgroup$
















                  4












                  4








                  4





                  $begingroup$

                  The flaw is in changing the integration interval.



                  $$-1le xle1iff frac1xle-1lor frac1xge 1$$






                  share|cite|improve this answer









                  $endgroup$



                  The flaw is in changing the integration interval.



                  $$-1le xle1iff frac1xle-1lor frac1xge 1$$







                  share|cite|improve this answer












                  share|cite|improve this answer



                  share|cite|improve this answer










                  answered yesterday









                  Yves DaoustYves Daoust

                  133k676232




                  133k676232






























                      draft saved

                      draft discarded




















































                      Thanks for contributing an answer to Mathematics Stack Exchange!


                      • Please be sure to answer the question. Provide details and share your research!

                      But avoid



                      • Asking for help, clarification, or responding to other answers.

                      • Making statements based on opinion; back them up with references or personal experience.


                      Use MathJax to format equations. MathJax reference.


                      To learn more, see our tips on writing great answers.




                      draft saved


                      draft discarded














                      StackExchange.ready(
                      function () {
                      StackExchange.openid.initPostLogin('.new-post-login', 'https%3a%2f%2fmath.stackexchange.com%2fquestions%2f3188675%2fwhat-is-wrong-with-this-false-proof-of-pi-0%23new-answer', 'question_page');
                      }
                      );

                      Post as a guest















                      Required, but never shown





















































                      Required, but never shown














                      Required, but never shown












                      Required, but never shown







                      Required, but never shown

































                      Required, but never shown














                      Required, but never shown












                      Required, but never shown







                      Required, but never shown







                      Popular posts from this blog

                      How did Captain America manage to do this?

                      迪纳利

                      南乌拉尔铁路局